• Anúncio Global
    Respostas
    Exibições
    Última mensagem

Derivada, velocidade e aceleração

Derivada, velocidade e aceleração

Mensagempor Janoca » Ter Jun 24, 2014 17:08

Nessa questão o problema foram as letras b, d ,e. Acredito q a letra a e c estão corretas.
Preciso entender o comportamento dessa equação.


A posição de uma partícula que se desloca ao longo do eixo x varia com o tempo segundo a equação x=\frac{{v}_{0}}{k}(1-{e}^{-kt}), t\geq0 onde {v}_{0} e k são constantes estritamente positivas.

a) Qual a velocidade no instante t?

resposta: v(t)={v}_{0}{e}^{-kt}

b) Com argumentos físicos, justifique a afirmação: "a função é estritamente crescente".

c) Qual a aceleração no instante t?

resposta: a(t)=-k{v}_{0}{e}^{-kt}

d) Com argumentos físicos, justifique a afirmação: " o gráfico da função tem a concavidade voltada para baixo".


e) Calcule o \lim_{t\rightarrow+\infty}\frac{{v}_{0}}{k}(1-{e}^{-kt}).
Janoca
Usuário Dedicado
Usuário Dedicado
 
Mensagens: 30
Registrado em: Sex Jun 06, 2014 16:47
Formação Escolar: GRADUAÇÃO
Área/Curso: matemática
Andamento: cursando

Re: Derivada, velocidade e aceleração

Mensagempor Janoca » Ter Jun 24, 2014 18:45

Eu ja calculei o Limite da letra e, segue abaixo:

\lim_{t\rightarrow+\infty}\frac{{v}_{0}}{k}(1-{e}^{-kt})= \frac{{v}_{0}}{k}, resta saber as alternativas b e d
Janoca
Usuário Dedicado
Usuário Dedicado
 
Mensagens: 30
Registrado em: Sex Jun 06, 2014 16:47
Formação Escolar: GRADUAÇÃO
Área/Curso: matemática
Andamento: cursando


Voltar para Cálculo: Limites, Derivadas e Integrais

 



  • Tópicos relacionados
    Respostas
    Exibições
    Última mensagem

Quem está online

Usuários navegando neste fórum: Nenhum usuário registrado e 29 visitantes

 



Assunto: cálculo de limites
Autor: Hansegon - Seg Ago 25, 2008 11:29

Bom dia.

Preciso de ajuda na solução deste problema, pois só chego ao resultado de 0 sobre 0.
Obrigado

\lim_{x\rightarrow-1} x³ +1/x²-1[/tex]


Assunto: cálculo de limites
Autor: Molina - Seg Ago 25, 2008 13:25

\lim_{x\rightarrow-1} \frac{{x}^{3}+1}{{x}^{2}-1}

Realmente se você jogar o -1 na equação dá 0 sobre 0.
Indeterminações deste tipo você pode resolver por L'Hôpital
que utiliza derivada.
Outro modo é transformar o numerador e/ou denominador
para que não continue dando indeterminado.

Dica: dividir o numerador e o denominador por algum valor é uma forma que normalmente dá certo. :y:

Caso ainda não tenha dado uma :idea:, avisa que eu resolvo.

Bom estudo!


Assunto: cálculo de limites
Autor: Guill - Dom Abr 08, 2012 16:03

\lim_{x\rightarrow-1}\frac{x^3+1}{x^2-1}

\lim_{x\rightarrow-1}\frac{(x+1)(x^2-x+1)}{(x+1)(x-1)}

\lim_{x\rightarrow-1}\frac{(x^2-x+1)}{(x-1)}=\frac{-3}{2}